Finding the theoretical bound of local spikes in an array - algorithm

You are given an arrayA[1..n], which consists of randomly permuted distinct integers.
An element of this array,A[i], is said to be a local spike, if it is larger than all of its preceding elements (in other words, for all j < i,A[i]> A[j]).
Show that the expected number of local spikes in A is O(logn).
If anybody can give me pointers to this question, it would be much appreciated!

It is similar to the reasoning about the quicksort time complexity.
So even though it is more about statistics, it can serve as a nice example of reasoning about algorithm complexity. Maybe it would be more suited to the CS stackexchange than statistics? That being said let's dive into the rabbit hole.
First, since all the numbers are distinct, we can ommit the part about array of random integers and simply take the integers 1, 2, ..., N without a loss of generality.
Now we can change the way of looking at the problem. Instead of having the array we can say that we are choosing a random number from the range 1..N without repetition.
Another observation is, that by choosing a number X, regardless of it being a local spike or not, we are disqualifying all the numbers that are lower from ever being a local spike.
Since we are now choosing the numbers, we can thus discard all Y, where Y < X from the candidate pool. This can be done since regardless of the position for a number lower than the spike, nothing will change for the subsequent spikes. Spike always has to be bigger than the maximum of the previous elements.
So the question becomes how many times can we repeat this procedure of:
Select a number from the pool of candidates as a new spike
Discard all the lower numbers
Before we discard whole candidate pool(starting with the full 1..N range). Not surprisingly, this is almost the same as the expected depth of the quicksort's recursion which is log(n).
A quick explanation if you don't want to check the wiki: Most of the time, we will discard ~half of the candidates. Sometimes less, sometimes more, however in the long run, the half is rather good estimate. More in depth explanation can be found here.

An elegant way to determine the solution to this problem is the following:
Define binary random variables X1, X2, ..., Xn by
Xi = 1 if A[i] is a local spike
Xi = 0 if A[i] is not a local spike
We see that the total number of local spikes is always the sum of the Xi. And we know that
E[X1 + X2 + ... + Xi] = E[X1] + E[X2] + ... + E[Xn]
By the linearity of expectation. So we must now turn out attention to deducing E[Xi] for each i.
Now E[Xi] = P(A[i] is a spike). What is the probability that A[i] > A[j] for all j < i?
This is just the probability that the maximum element of A[1], A[2], ..., A[i] is A[i]. But this maximum element could be located anywhere from A[1] to A[i] with equal probability. So the probability is 1/i that the maximum element is A[i].
So E[Xi] = 1/i. Then we see that
E[total number of spikes] = E[X1] + E[X2] + ... + E[Xn] = 1/1 + 1/2 + ... + 1/n
This is the nth harmonic number, Hn. And it is well known that Hn ~ ln(n). This is because ln(n) <= Hn <= ln(n) + 1 for all n (easy proof involving Riemann sums, but requires a smidge of calculus). This shows that there are O(log n) spikes, on average.

Related

How many times variable m is updated

Given the following pseudo-code, the question is how many times on average is the variable m being updated.
A[1...n]: array with n random elements
m = a[1]
for I = 2 to n do
if a[I] < m then m = a[I]
end for
One might answer that since all elements are random, then the variable will be updated on average on half the number of iterations of the for loop plus one for the initialization.
However, I suspect that there must be a better (and possibly the only correct) way to prove it using binomial distribution with p = 1/2. This way, the average number of updates on m would be
M = 1 + Σi=1 to n-1[k.Cn,k.pk.(1-p)(n-k)]
where Cn,k is the binomial coefficient. I have tried to solve this but I have stuck some steps after since I do not know how to continue.
Could someone explain me which of the two answers is correct and if it is the second one, show me how to calculate M?
Thank you for your time
Assuming the elements of the array are distinct, the expected number of updates of m is the nth harmonic number, Hn, which is the sum of 1/k for k ranging from 1 to n.
The summation formula can also be represented by the recursion:
H1 &equals; 1
Hn &equals; Hn−1&plus;1/n (n > 1)
It's easy to see that the recursion corresponds to the problem.
Consider all permutations of n−1 numbers, and assume that the expected number of assignments is Hn−1. Now, every permutation of n numbers consists of a permutation of n−1 numbers, with a new smallest number inserted in one of n possible insertion points: either at the beginning, or after one of the n−1 existing values. Since it is smaller than every number in the existing series, it will only be assigned to m in the case that it was inserted at the beginning. That has a probability of 1/n, and so the expected number of assignments of a permutation of n numbers is Hn−1 + 1/n.
Since the expected number of assignments for a vector of length one is obviously 1, which is H1, we have an inductive proof of the recursion.
Hn is asymptotically equal to ln n &plus; γ where γ is the Euler-Mascheroni constant, approximately 0.577. So it increases without limit, but quite slowly.
The values for which m is updated are called left-to-right maxima, and you'll probably find more information about them by searching for that term.
I liked #rici answer so I decided to elaborate its central argument a little bit more so to make it clearer to me.
Let H[k] be the expected number of assignments needed to compute the min m of an array of length k, as indicated in the algorithm under consideration. We know that
H[1] = 1.
Now assume we have an array of length n > 1. The min can be in the last position of the array or not. It is in the last position with probability 1/n. It is not with probability 1 - 1/n. In the first case the expected number of assignments is H[n-1] + 1. In the second, H[n-1].
If we multiply the expected number of assignments of each case by their probabilities and sum, we get
H[n] = (H[n-1] + 1)*1/n + H[n-1]*(1 - 1/n)
= H[n-1]*1/n + 1/n + H[n-1] - H[n-1]*1/n
= 1/n + H[n-1]
which shows the recursion.
Note that the argument is valid if the min is either in the last position or in any the first n-1, not in both places. Thus we are using that all the elements of the array are different.

Place "sum" and "multiply" operators between the elements of a given list of integers so that the expression results in a specified value

I was given a tricky question.
Given:
A = [a1,a2,...an] (list of positive integers with length "n")
r (positive integer)
Find a list of { *, + } operators
O = [o1,o2,...on-1]
so that if we placed those operators between the elements of "A", the resulting expression would evaluate to "r". Only one solution is required.
So for example if
A = [1,2,3,4]
r = 14
then
O = [*, +, *]
I've implemented a simple recursive solution with some optimisation, but of course it's exponential O(2^n) time, so for an input with length 40, it works for ages.
I wanted to ask if any of you know a sub-exponential solution for this?
Update
Elements of A are between 0-10000,
r can be arbitrarily big
Let A and B be positive integers. Then A + B ≤ A × B + 1.
This little fact can be used to construct a very efficient algorithm.
Let's define a graph. The graph nodes correspond to operations lists, for example, [+, ×, +, +, ×]. There is an edge from graph node X to graph node Y if the Y can be obtained by changing a single + to a × in X. The graph has a source at the node corresponding to [+, +, ..., +].
Now perform a breadth-first search from the source node, constructing the graph as you go. When expanding a node [+, ×, +, +, ×], for example, you (optionally construct then) connect to the nodes [×, ×, +, +, ×], [+, ×, ×, +, ×], and [+, ×, +, ×, ×]. Do not expand to a node if the result of evaluating it is greater than r + k(O), where k(O) is the number of +'s in the operation list O. This is because of the "+ 1" in the fact at the beginning of the answer - consider the case of a = [1, 1, 1, 1, 1], r = 1.
This approach uses O(n 2n) time and O(2n) space (where both are potentially very-loose worst case bounds). This is still an exponential algorithm, however I think you will find it performs very reasonably for non-sinister inputs. (I suspect this problem is NP-complete, which is why I am happy with this "non-sinister inputs" escape clause.)
Here's an O(rn^2)-time, O(rn)-space DP approach. If r << 2^n then this will have better worst-case behaviour than exponential-time branch-and-bound approaches, though even then the latter may still be faster on many instances. This is pseudo-polynomial time, because it takes time proportional to the value of part of its input (r), not its size (which would be log2(r)). Specifically it needs rn bits of memory, so it should give answers in a few seconds for up to around rn < 1,000,000,000 and n < 1000 (e.g. n = 100, r = 10,000,000).
The key observation is that any formula involving all n numbers has a final term that consists of some number i of factors, where 1 <= i <= n. That is, any formula must be in one of the following n cases:
(a formula on the first n-1 terms) + a[n]
(a formula on the first n-2 terms) + a[n-1] * a[n]
(a formula on the first n-3 terms) + a[n-2] * a[n-1] * a[n]
...
a[1] * a[2] * ... * a[n]
Let's call the "prefix" of a[] consisting of the first i numbers P[i]. If we record, for each 0 <= i <= n-1, the complete set of values <= r that can be reached by some formula on P[i], then based on the above, we can quite easily compute the complete set of values <= r that can be reached by P[n]. Specifically, let X[i][j] be a true or false value that indicates whether the prefix P[i] can achieve the value j. (X[][] could be stored as an array of n size-(r+1) bitmaps.) Then what we want to do is compute X[n][r], which will be true if r can be reached by some formula on a[], and false otherwise. (X[n][r] isn't quite the full answer yet, but it can be used to get the answer.)
X[1][a[1]] = true. X[1][j] = false for all other j. For any 2 <= i <= n and 0 <= j <= r, we can compute X[i][j] using
X[i][j] = X[i - 1][j - a[i]] ||
X[i - 2][j - a[i-1]*a[i]] ||
X[i - 3][j - a[i-2]*a[i-1]*a[i]] ||
... ||
X[1][j - a[2]*a[3]*...*a[i]] ||
(a[1]*a[2]*...*a[i] == j)
Note that the last line is an equality test that compares the product of all i numbers in P[i] to j, and returns true or false. There are i <= n "terms" (rows) in the expression for X[i][j], each of which can be computed in constant time (note in particular that the multiplications can be built up in constant time per row), so computing a single value X[i][j] can be done in O(n) time. To find X[n][r], we need to calculate X[i][j] for every 1 <= i <= n and every 0 <= j <= r, so there is O(rn^2) overall work to do. (Strictly speaking we may not need to compute all of these table entries if we use memoization instead of a bottom-up approach, but many inputs will require us to compute a large fraction of them anyway, so it's likely that the latter is faster by a small constant factor. Also a memoization approach requires keeping an "already processed" flag for each DP cell -- which doubles the memory usage when each cell is just 1 bit!)
Reconstructing a solution
If X[n][r] is true, then the problem has a solution (satisfying formula), and we can reconstruct one in O(n^2) time by tracing back through the DP table, starting from X[n][r], at each location looking for any term that enabled the current location to assume the value "true" -- that is, any true term. (We could do this reconstruction step faster by storing more than a single bit per (i, j) combination -- but since r is allowed to be "arbitrarily big", and this faster reconstruction won't improve the overall time complexity, it probably makes more sense to go with the approach that uses the fewest bits per DP table entry.) All satisfying solutions can be reconstructed this way, by backtracking through all true terms instead of just picking any one -- but there may be an exponential number of them.
Speedups
There are two ways that calculation of an individual X[i][j] value can be sped up. First, because all the terms are combined with ||, we can stop as soon as the result becomes true, since no later term can make it false again. Second, if there is no zero anywhere to the left of i, we can stop as soon as the product of the final numbers becomes larger than r, since there's no way for that product to be decreased again.
When there are no zeroes in a[], that second optimisation is likely to be very important in practice: it has the potential to make the inner loop much smaller than the full i-1 iterations. In fact if a[] contains no zeroes, and its average value is v, then after k terms have been computed for a particular X[i][j] value the product will be around v^k -- so on average, the number of inner loop iterations (terms) needed drops from n to log_v(r) = log(r)/log(v). That might be much smaller than n, in which case the average time complexity for this model drops to O(rn*log(r)/log(v)).
[EDIT: We actually can save multiplications with the following optimisation :)]
8/32/64 X[i][j]s at a time: X[i][j] is independent of X[i][k] for k != j, so if we are using bitsets to store these values, we can calculate 8, 32 or 64 of them (or maybe more, with SSE2 etc.) in parallel using simple bitwise OR operations. That is, we can calculate the first term of X[i][j], X[i][j+1], ..., X[i][j+31] in parallel, OR them into the results, then calculate their second terms in parallel and OR them in, etc. We still need to perform the same number of subtractions this way, but the products are all the same, so we can reduce the number of multiplications by a factor of 8/32/64 -- as well as, of course, the number of memory accesses. OTOH, this makes the first optimisation from the previous paragraph harder to accomplish -- you have to wait until an entire block of 8/32/64 bits have become true before you can stop iterating.
Zeroes: Zeroes in a[] may allow us to stop early. Specifically, if we have just computed X[i][r] for some i < n and found it to be true, and there is a zero anywhere to the right of position i in a[], then we can stop: we already have a formula on the first i numbers that evaluates to r, and we can use that zero to "kill off" all numbers to the right of position i by creating one big product term that includes all of them.
Ones: An interesting property of any a[] entry containing the value 1 is that it can be moved to any other position in a[] without affecting whether or not there is a solution. This is because every satisfying formula either has a * on at least one side of this 1, in which case it multiplies some other term and has no effect there, and would likewise have no effect anywhere else; or it has a + on both sides (imagine extra + signs before the first position and after the last), in which case it might as well be added in anywhere.
So, we can safely shunt all 1 values to the end of a[] before doing anything else. The point of doing this is that now we don't have to evaluate these rows of X[][] at all, because they only influence the outcome in a very simple way. Suppose there are m < n ones in a[], which we have moved to the end. Then after computing the m+1 values X[n-m][r-m], X[n-m][r-m+1], X[n-m][r-m+2], ..., X[n-m][r], we already know what X[n][r] must be: if any of them are true, then X[n][r] must be true, otherwise (if they are all false) it must be false. This is because the final m ones can add anywhere from 0 up to m to a formula on the first n-m values. (But if a[] consists entirely of 1s, then at least 1 must be "added" -- they can't all multiply some other term.)
Here is another approach that might be helpful. It is sometimes known as a "meet-in-the-middle" algorithm and runs in O(n * 2^(n/2)). The basic idea is this. Suppose n = 40 and you know that the middle slot is a +. Then, you can brute force all N := 2^20 possibilities for each side. Let A be a length N array storing the possible values of the left side, and similarly let B be a length N array storing the values for the right side.
Then, after sorting A and B, it is not hard to efficiently check for whether any two of them sum to r (e.g. for each value in A, do a binary search on B, or you can even do it in linear time if both arrays are sorted). This part takes O(N * log N) = O(n * 2^(n/2)) time.
Now, this was all assuming the middle slot is a +. If not, then it has to be a *, and you can combine the middle two elements into one (their product), reducing the problem to n = 39. Then you try the same thing, and so on. If you analyze it carefully, you should get O(n * 2^(n/2)) as the asymptotic complexity, since actually the largest term dominates.
You need to do some bookkeeping to actually recover the +'s and *'s, which I have left out to simplify the explanation.

random merge sort

I was given the following question in an algorithms book:
Suppose a merge sort is implemented to split a file at a random position, rather then exactly in the middle. How many comparisons would be used by such method to sort n elements on average?
Thanks.
To guide you to the answer, consider these more specific questions:
Assume the split is always at 10%, or 25%, or 75%, or 90%. In each case: what's the impact on recursion depths? How many comparisons need to be per recursion level?
I'm partially agree with #Armen, they should be comparable.
But: consider the case when they are split in the middle. To merge two lists of lengths n we would need 2*n - 1 comparations (sometimes less, but we'll consider it fixed for simplicity), each of them producing the next value. There would be log2(n) levels of merges, that gives us approximately n*log2(n) comparations.
Now considering the random-split case: The maximum number of comparations needed to merge a list of length n1 with one of length n2 will be n1 + n2 - 1. Howerer, the average number will be close to it, because even for the most unhappy split 1 and n-1 we'll need an average of n/2 comparations. So we can consider that the cost of merging per level will be the same as in even case.
The difference is that in random case the number of levels will be larger, and we can consider that n for next level would be max(n1, n2) instead of n/2. This max(n1, n2) will tend to be 3*n/4, that gives us the approximate formula
n*log43(n) // where log43 is log in base 4/3
that gives us
n * log2(n) / log2(4/3) ~= 2.4 * n * log2(n)
This result is still larger than the correct one because we ignored that the small list will have fewer levels, but it should be close enough. I suppose that the correct answer will be the number of comparations on average will double
You can get an upper bound of 2n * H_{n - 1} <= 2n ln n using the fact that merging two lists of total length n costs at most n comparisons. The analysis is similar to that of randomized quicksort (see http://www.cs.cmu.edu/afs/cs/academic/class/15451-s07/www/lecture_notes/lect0123.pdf).
First, suppose we split a list of length n into 2 lists L and R. We will charge the first element of R for a comparison against all of the elements of L, and the last element of L for a comparison against all elements of R. Even though these may not be the exact comparisons that are executed, the total number of comparisons we are charging for is n as required.
This handles one level of recursion, but what about the rest? We proceed by concentrating only on the "right-to-left" comparisons that occur between the first element of R and every element of L at all levels of recursion (by symmetry, this will be half the actual expected total). The probability that the jth element is compared to the ith element is 1/(j - i) where j > i. To see this, note that element j is compared with element i exactly when it is the first element chosen as a "splitting element" from among the set {i + 1,..., j}. That is, elements i and j are split into two lists as soon as the list they are in is split at some element from {i + 1,..., j}, and element j is charged for a comparison with i exactly when element j is the element that is chosen from this set.
Thus, the expected total number of comparisons involving j is at most H_n (i.e., 1 + 1/2 + 1/3..., where the number of terms is at most n - 1). Summing across all possible j gives n * H_{n - 1}. This only counted "right-to-left" comparisons, so the final upper bound is 2n * H_{n - 1}.

Greatest GCD between some numbers

We've got some nonnegative numbers. We want to find the pair with maximum gcd. actually this maximum is more important than the pair!
For example if we have:
2 4 5 15
gcd(2,4)=2
gcd(2,5)=1
gcd(2,15)=1
gcd(4,5)=1
gcd(4,15)=1
gcd(5,15)=5
The answer is 5.
You can use the Euclidean Algorithm to find the GCD of two numbers.
while (b != 0)
{
int m = a % b;
a = b;
b = m;
}
return a;
If you want an alternative to the obvious algorithm, then assuming your numbers are in a bounded range, and you have plenty of memory, you can beat O(N^2) time, N being the number of values:
Create an array of a small integer type, indexes 1 to the max input. O(1)
For each value, increment the count of every element of the index which is a factor of the number (make sure you don't wraparound). O(N).
Starting at the end of the array, scan back until you find a value >= 2. O(1)
That tells you the max gcd, but doesn't tell you which pair produced it. For your example input, the computed array looks like this:
1 2 3 4 5 6 7 8 9 10 11 12 13 14 15
4 2 1 1 2 0 0 0 0 0 0 0 0 0 1
I don't know whether this is actually any faster for the inputs you have to handle. The constant factors involved are large: the bound on your values and the time to factorise a value within that bound.
You don't have to factorise each value - you could use memoisation and/or a pregenerated list of primes. Which gives me the idea that if you are memoising the factorisation, you don't need the array:
Create an empty set of int, and a best-so-far value 1.
For each input integer:
if it's less than or equal to best-so-far, continue.
check whether it's in the set. If so, best-so-far = max(best-so-far, this-value), continue. If not:
add it to the set
repeat for all of its factors (larger than best-so-far).
Add/lookup in a set could be O(log N), although it depends what data structure you use. Each value has O(f(k)) factors, where k is the max value and I can't remember what the function f is...
The reason that you're finished with a value as soon as you encounter it in the set is that you've found a number which is a common factor of two input values. If you keep factorising, you'll only find smaller such numbers, which are not interesting.
I'm not quite sure what the best way is to repeat for the larger factors. I think in practice you might have to strike a balance: you don't want to do them quite in decreasing order because it's awkward to generate ordered factors, but you also don't want to actually find all the factors.
Even in the realms of O(N^2), you might be able to beat the use of the Euclidean algorithm:
Fully factorise each number, storing it as a sequence of exponents of primes (so for example 2 is {1}, 4 is {2}, 5 is {0, 0, 1}, 15 is {0, 1, 1}). Then you can calculate gcd(a,b) by taking the min value at each index and multiplying them back out. No idea whether this is faster than Euclid on average, but it might be. Obviously it uses a load more memory.
The optimisations I can think of is
1) start with the two biggest numbers since they are likely to have most prime factors and thus likely to have the most shared prime factors (and thus the highest GCD).
2) When calculating the GCDs of other pairs you can stop your Euclidean algorithm loop if you get below your current greatest GCD.
Off the top of my head I can't think of a way that you can work out the greatest GCD of a pair without trying to work out each pair individually (and optimise a bit as above).
Disclaimer: I've never looked at this problem before and the above is off the top of my head. There may be better ways and I may be wrong. I'm happy to discuss my thoughts in more length if anybody wants. :)
There is no O(n log n) solution to this problem in general. In fact, the worst case is O(n^2) in the number of items in the list. Consider the following set of numbers:
2^20 3^13 5^9 7^2*11^4 7^4*11^3
Only the GCD of the last two is greater than 1, but the only way to know that from looking at the GCDs is to try out every pair and notice that one of them is greater than 1.
So you're stuck with the boring brute-force try-every-pair approach, perhaps with a couple of clever optimizations to avoid doing needless work when you've already found a large GCD (while making sure that you don't miss anything).
With some constraints, e.g the numbers in the array are within a given range, say 1-1e7, it is doable in O(NlogN) / O(MAX * logMAX), where MAX is the maximum possible value in A.
Inspired from the sieve algorithm, and came across it in a Hackerrank Challenge -- there it is done for two arrays. Check their editorial.
find min(A) and max(A) - O(N)
create a binary mask, to mark which elements of A appear in the given range, for O(1) lookup; O(N) to build; O(MAX_RANGE) storage.
for every number a in the range (min(A), max(A)):
for aa = a; aa < max(A); aa += a:
if aa in A, increment a counter for aa, and compare it to current max_gcd, if counter >= 2 (i.e, you have two numbers divisible by aa);
store top two candidates for each GCD candidate.
could also ignore elements which are less than current max_gcd;
Previous answer:
Still O(N^2) -- sort the array; should eliminate some of the unnecessary comparisons;
max_gcd = 1
# assuming you want pairs of distinct elements.
sort(a) # assume in place
for ii = n - 1: -1 : 0 do
if a[ii] <= max_gcd
break
for jj = ii - 1 : -1 :0 do
if a[jj] <= max_gcd
break
current_gcd = GCD(a[ii], a[jj])
if current_gcd > max_gcd:
max_gcd = current_gcd
This should save some unnecessary computation.
There is a solution that would take O(n):
Let our numbers be a_i. First, calculate m=a_0*a_1*a_2*.... For each number a_i, calculate gcd(m/a_i, a_i). The number you are looking for is the maximum of these values.
I haven't proved that this is always true, but in your example, it works:
m=2*4*5*15=600,
max(gcd(m/2,2), gcd(m/4,4), gcd(m/5,5), gcd(m/15,15))=max(2, 2, 5, 5)=5
NOTE: This is not correct. If the number a_i has a factor p_j repeated twice, and if two other numbers also contain this factor, p_j, then you get the incorrect result p_j^2 insted of p_j. For example, for the set 3, 5, 15, 25, you get 25 as the answer instead of 5.
However, you can still use this to quickly filter out numbers. For example, in the above case, once you determine the 25, you can first do the exhaustive search for a_3=25 with gcd(a_3, a_i) to find the real maximum, 5, then filter out gcd(m/a_i, a_i), i!=3 which are less than or equal to 5 (in the example above, this filters out all others).
Added for clarification and justification:
To see why this should work, note that gcd(a_i, a_j) divides gcd(m/a_i, a_i) for all j!=i.
Let's call gcd(m/a_i, a_i) as g_i, and max(gcd(a_i, a_j),j=1..n, j!=i) as r_i. What I say above is g_i=x_i*r_i, and x_i is an integer. It is obvious that r_i <= g_i, so in n gcd operations, we get an upper bound for r_i for all i.
The above claim is not very obvious. Let's examine it a bit deeper to see why it is true: the gcd of a_i and a_j is the product of all prime factors that appear in both a_i and a_j (by definition). Now, multiply a_j with another number, b. The gcd of a_i and b*a_j is either equal to gcd(a_i, a_j), or is a multiple of it, because b*a_j contains all prime factors of a_j, and some more prime factors contributed by b, which may also be included in the factorization of a_i. In fact, gcd(a_i, b*a_j)=gcd(a_i/gcd(a_i, a_j), b)*gcd(a_i, a_j), I think. But I can't see a way to make use of this. :)
Anyhow, in our construction, m/a_i is simply a shortcut to calculate the product of all a_j, where j=1..1, j!=i. As a result, gcd(m/a_i, a_i) contains all gcd(a_i, a_j) as a factor. So, obviously, the maximum of these individual gcd results will divide g_i.
Now, the largest g_i is of particular interest to us: it is either the maximum gcd itself (if x_i is 1), or a good candidate for being one. To do that, we do another n-1 gcd operations, and calculate r_i explicitly. Then, we drop all g_j less than or equal to r_i as candidates. If we don't have any other candidate left, we are done. If not, we pick up the next largest g_k, and calculate r_k. If r_k <= r_i, we drop g_k, and repeat with another g_k'. If r_k > r_i, we filter out remaining g_j <= r_k, and repeat.
I think it is possible to construct a number set that will make this algorithm run in O(n^2) (if we fail to filter out anything), but on random number sets, I think it will quickly get rid of large chunks of candidates.
pseudocode
function getGcdMax(array[])
arrayUB=upperbound(array)
if (arrayUB<1)
error
pointerA=0
pointerB=1
gcdMax=0
do
gcdMax=MAX(gcdMax,gcd(array[pointera],array[pointerb]))
pointerB++
if (pointerB>arrayUB)
pointerA++
pointerB=pointerA+1
until (pointerB>arrayUB)
return gcdMax

Programing Pearls - Random Select algorithm

Page 120 of Programming Pearls 1st edition presents this algorithm for selecting M equally probable random elements out of a population of N integers.
InitToEmpty
Size := 0
While Size < M do
T := RandInt(1,N)
if not Member(T)
Insert(T)
Size := Size + 1
It is stated that the expected number of Member tests is less than 2M, as long as M < N/2.
I'd like to know how to prove it, but my algorithm analysis background is failing me.
I understand that the closer M is to N, the longer the program will take, because the result set will have more elements and the likelihood of RandInt selecting an existing one will increase proportionally.
Can you help me figuring out this proof?
I am not a math wizard, but I will give it a rough shot. This is NOT guaranteed to be right though.
For each additional member of M, you pick a number, see if it's there, and if is add it. Otherwise, you try again. Trying something until you're successful is called a geometric probability distribution.
http://en.wikipedia.org/wiki/Geometric_distribution
So you are running M geometric trials. Each trial has expected value 1/p, so will take expected 1/p tries to get a number not already in M. p is N minus the number of numbers we've already added from M divided by N (i.e. how many unpicked items / total items). So for the fourth number, p = (N -3) / N, which is the probability of picking an unused number, so the expected number of picks for the third number is N / N-3 .
The expected value of the run time is all of these added together. So something like
E(run time) = N/N + N/(N -1) + N/(N -2 ) ... + N/ (N-M)
Now if M < N/2, then the last element in that summation is bounded above by 2. ((N/N/2) == 2)). It's also obviously the largest element in the whole summation. So if the biggest element is two picks, and there are M elements being summed, the EV of the whole run time is bounded above by 2M.
Ask me if any of this is unclear. Correct me if any of this is wrong :)
Say we have chosen K elements out of N. Then our next try has probability (N-K)/N of succeeding, so the number of tries that it takes to find the K + 1 st element is geometrically distributed with mean N/(N-K).
So if 2M < N we expect it to take less than two tries to get each element.

Resources